0% found this document useful (0 votes)
604 views7 pages

Advanced Math Problems & Solutions

The document contains solutions to 6 math problems labeled A1 through A6. The summaries are: 1) A cube is inscribed in a cone with radius 1 base and height 3. The side length of the cube is calculated to be √7/3. 2) It is shown that the sum of the areas of two regions defined by an arc on a unit circle depends only on the arc length, not the position of the arc. 3) Using calculus techniques, it is proved that for a function with a continuous third derivative, there exists a point where the function and its first three derivatives have the same sign.

Uploaded by

Abir Mahmud
Copyright
© © All Rights Reserved
We take content rights seriously. If you suspect this is your content, claim it here.
Available Formats
Download as PDF, TXT or read online on Scribd
0% found this document useful (0 votes)
604 views7 pages

Advanced Math Problems & Solutions

The document contains solutions to 6 math problems labeled A1 through A6. The summaries are: 1) A cube is inscribed in a cone with radius 1 base and height 3. The side length of the cube is calculated to be √7/3. 2) It is shown that the sum of the areas of two regions defined by an arc on a unit circle depends only on the arc length, not the position of the arc. 3) Using calculus techniques, it is proved that for a function with a continuous third derivative, there exists a point where the function and its first three derivatives have the same sign.

Uploaded by

Abir Mahmud
Copyright
© © All Rights Reserved
We take content rights seriously. If you suspect this is your content, claim it here.
Available Formats
Download as PDF, TXT or read online on Scribd
You are on page 1/ 7

Putnam 1998 (Problems and Solutions)

A1. A right circular cone has base of radius 1 and height 3. A cube is inscribed
in the cone so that one face of the cube is contained in the base of the cone.
What is the side-length of the cube.
Solution. Consider the cross section obtained by slicing vertically by a plane
that contains a diagonal of the base of the
√ cube. We obtain a rectangle of height
s = side-length of the cube and width 2s. By similar triangles
3 s
= 1
√ ⇒
1 1− 2 2s
¡ √ ¢ √
6 6 3 2−2 9 2−6
s = √ =¡ √ ¢¡ √ ¢= .
3 2+2 3 2+2 3 2−2 7

A2. Let s be any arc of the unit circle lying entirely in the first quadrant. Let
A be the area of the region lying below s and above the x-axis and let B be the
area of the region lying to the right of the y-axis and to the left of s. Prove that
A + B depends only on the arc length of s and not on the position of s.

Solution. Let s run from θ 1 to θ2 . Then


Z cos θ1 p Z θ1
A = 1 − x2 dx = − sin2 θ dθ
cos θ2 θ2
Z sin θ2 p Z θ2
B = 2
1 − y dy = cos2 θ dθ
sin θ1 θ1

Note that

∂θ 1 (A + B) = − sin2 θ 1 − cos2 θ1 = −1 ⇒ A + B = −θ1 + f (θ2 )

while

∂θ2 (A + B) = sin2 θ2 + cos2 θ2 = 1 ⇒ A + B = θ 2 + g (θ1 ) .

Hence, since A + B = 0 when θ 1 = θ2 ,

A + B = θ2 − θ1 + C = θ2 − θ1 .

A3. Let f be a real function on the real line with continuous third derivative.
Prove that there exists a point a such that

f (a) · f 0 (a) · f 00 (a) · f 000 (a) ≥ 0.

1
Solution. Assume otherwise. Then
f (x) · f 0 (x) · f 00 (x) · f 000 (x) < 0.
In particular, each of the factors is never zero. By replacing f (x) by −f (x) if
necessary, we may assume that f (x) > 0, and by replacing f (x) by f (−x) if
necessary, we may assume that f 0 (x) > 0. There are two cases:
Case 1: f 00 (x) > 0 and f 000 (x) < 0.
Since f 000 (x) < 0, the graph of f 0 (x) is concave down and hence the graph
of f 0 (x) lies below its tangent line at x = 0. Thus,
µ 0 ¶
−f (0)
f 0 (x) ≤ f 0 (0) + f 00 (0) x and f 0 ≤ 0 (contradiction).
f 00 (0)
Case 2: f 00 (x) < 0 and f 000 (x) > 0.
Since f 00 (x) < 0, the graph of f (x) is concave down and hence the graph of
f (x) lies below its tangent line at x = 0. Thus,
µ ¶
−f (0)
f (x) ≤ f (0) + f 0 (0) x and f ≤ 0 (contradiction).
f 0 (0)

A4. Let A1 = 0 and let A2 = 1. For n > 2, the number An is defined by


concatenating the decimal expansions of An−1 and An−2 from left to right. For
example A3 = A2 A1 = 10, A4 = A3 A2 = 101, A5 = A4 A3 = 10110, and so
forth. Determine all n such that 11 divides An .
n
Solution. Since 10n ≡ (−1) mod 11, an integer is divisible by 11 iff the alter-
nating sum of its digits (from right to left so that the first term is positive) is 0
mod 11. Let en be the alternating sum of the digits mod 11 of An from right to
left, and let Fn be the number digits in An . Of course Fn is just the Fibonacci
F
sequence. Note that en+2 = (−1) n en+1 + en . We have Fn+2 = Fn+1 + Fn and
F1 = 1, and F2 = 1, we see that Fn is even iff n is divisible by 3. We have (all
mod 11)

en+2 = (−1)Fn en+1 + en


e3k+3 = (−1)F3k+1 e3k+2 + e3k+1 = −e3k+2 + e3k+1
e3k+4 = (−1)F3k+2 e3k+3 + e3k+2 = −e3k+3 + e3k+2 = − (−e3k+2 + e3k+1 ) + e3k+2
= −e3k+1 + 2e3k+2
e3k+5 = (−1)F3k+3 e3k+4 + e3k+3 = e3k+4 + e3k+3 = (−e3k+1 + 2e3k+2 ) + (−e3k+2 + e3k+1 )
= e3k+2
e3k+6 = (−1)F3k+4 e3k+5 + e3k+4 = −e3k+5 + e3k+4 = −e3k+2 + (−e3k+1 + 2e3k+2 )
= e3k+2 − e3k+1
e3k+7 = (−1)F3k+5 e3k+6 + e3k+5 = −e3k+6 + e3k+5 = − (e3k+2 − e3k+1 ) + e3k+2 = e3k+1
e3k+8 = (−1)F3k+6 e3k+7 + e3k+6 = e3k+7 + e3k+6 = e3k+1 + e3k+2 − e3k+1 = e3k+2
e3k+9 = (−1)F3k+7 e3k+8 + e3k+7 = −e3k+8 + e3k+7 = −e3k+2 + e3k+1 = e3k+3

2
Thus, en is periodic of period 6. Now
e1 = 0, e2 = 1, e3 = −e2 + e1 = −1,
e4 = −e1 + 2e2 = 2, e5 = e2 = 1, e6 = e2 − e1 = 1.
Thus, en is divisible by 11 iff n ≡ 1 mod 6.

A5. Let F be a finite collection of open disks in R2 whose union contains a


set E ⊆ R2 . Prove that there is a pairwise disjoint collection D1 , ..., Dn in F
such that
[n
3Dj ⊇ E.
j=1

Here, if D is the disc of radius r and center P , then 3D is the disc of radius 3r
and center P.

Solution. Note that if D and D0 are open disks of radius r and r0 , with r ≤ r0 ,
then D ∩ D0 6= φ ⇒ 3D0 ⊇ D. Starting with a largest disk, color it (say, red),
and remove the disks which intersect it. The 3-fold enlargement of the colored
disk together with the remaining disks will still cover E, by the above. Then
color a largest remaining uncolored disk, and remove the remaining uncolored
disks which intersect it. Continuing, the process eventually stops, since there
are a finite number of disks. Moreover, at each stage the 3-fold enlargements of
the colored disks and the remaining uncolored disks cover E. At the end of the
process, no uncolored disks remain, and the 3-fold enlargements of the colored
disks cover E. The colored disks are pairwise disjoint by construction.

A6. Let A, B and C denote distinct points with integer coordinates in R2 .


Prove that if
2
(|AB| + |BC|) < 8 · [ABC] + 1
then A, B, C are three vertices of a square. Here |XY | is the length of the
segment XY and [ABC] is the area of triangle ABC.

Solution. We have [ABC] = 12 |AB| |BC| sin θ, where θ is the angle of triangle
ABC at vertex B. Thus, 4 · [ABC] ≤ 2 |AB| |BC| with equality only if θ = 90◦ .
Also 2 |AB| |BC| ≤ |AB|2 + |BC|2 with equality only if |AB| = |BC|, since
2
0 ≤ (|AB| − |BC|) . Hence,
8 · [ABC] ≤ 4 · [ABC] + 2 |AB| |BC|
≤ 4 · [ABC] + |AB|2 + |BC|2
≤ 2 |AB| |BC| + |AB|2 + |BC|2
= (|AB| + |BC|)2 < 8 · [ABC] + 1
2 2
The intermediate expression 4 · [ABC] + |AB| + |BC| is an integer, since
2 · [ABC] is a determinant of a 2 × 2 matrix with integer coefficients. Thus, we
have all equalities, and θ = 90◦ and |AB| = |BC| .

3
B1. Find the minimum value of
6 ¡ ¢
(x + 1/x) − x6 + 1/x6 − 2
(x + 1/x)3 + (x3 + 1/x3 )

Solution. We have
¡ ¢ ¡ ¢2
(x + 1/x)6 − x6 + 1/x6 − 2 6
(x + 1/x) − x3 + 1/x3
=
(x + 1/x)3 + (x3 + 1/x3 ) (x + 1/x)3 + (x3 + 1/x3 )
³ ¡ ¢´ ³ ¡ ¢´
(x + 1/x)3 − x3 + 1/x3 3
(x + 1/x) + x3 + 1/x3
=
(x + 1/x)3 + (x3 + 1/x3 )
¡ ¢
= (x + 1/x)3 − x3 + 1/x3
= 3 (x + 1/x)

d x2 − 1
dx (x + 1/x) = = 0, x > 0 ⇒ x = 1.
x2
As x + 1/x approaches ∞ as x → +∞ and x → 0+ , there is a minimum and
the only candidate is x = 1. The minimum value is 3 (1 + 1/1) = 6.

B2. Given a point (a, b) with 0 < b < a, determine the minimum perimeter of
a triangle with one vertex at (a, b) one on the x-axis and one on the line y = x.
You may assume that a triangle with minimum perimeter exists.

Solution. Note that the distance of any point (d, d) on the line y = x to the
point (a, b) is the same as the distance of (d, d) to (b, a) , the reflection of (a, b)
in the line y = x. Also, the distance of any point (c, 0) to (a, b) is the same
as the distance of (c, 0) to (a, −b) , the reflection of (a, b) in the x-axis. Thus,
the perimeter of the triangle (d, d) , (a, b) , (c, 0) is the same as the broken line
segment with vertices (a, −b) , (c, 0) , (d, d) , (b, a). The length of this broken
line segment is no qgreater than the distance between the endpoints (b, a) and
2 2 p
(a, −b) , namely (b − a) + (a + b) = 2 (a2 + b2 ). We can choose c and
d
pso that the broken segment (a, −b) , (c, 0) , (d, d) , (b, a) is straight. Thus,
2 (a2 + b2 ) is the minimum possible perimeter.

B3. Let H be the unit hemisphere {(x, y, z) : x2 + y 2 + z 2 = 1, z ≥ 0}, C the


unit circle {(x, y, 0) : x2 + y2 = 1}, and P the regular pentagon inscribed in C.
Determine the surface area of that portion of H lying over the planar region
inside P , and write your answer in the form A sin α + B cos β, where A, B, α, β
are real numbers.

4
Solution. The desired area A is 12 the area of the sphere minus 5 polar caps
that each extends an angle of π5 from its pole. Thus,
à Z 2π Z π5 !
1
A = 4π − 5 sin ϕ dϕdθ
2 0 0
³ π ´
= 2π − 5 · π − cos + 1
5
π π π
= −3π + 5π cos = −3π sin + 5π cos .
5 2 5

B4. Find necessary and sufficient conditions on positive integers m and n so


that
mn−1
X
(−1)bi/mc+bi/nc = 0.
i=0

Solution. The number of terms in the sum is mn which is odd if both m and
n are odd. Thus, the sum (consisting of 1s and −1s) cannot be zero if both m
and n are odd. Suppose that m + n is odd (e.g., m is even and n is odd). In
this case we claim

(bi/mc + bi/nc) + (b(mn − 1 − i) /mc + b(mn − 1 − i) /nc) = m + n − 2 (1)

which is odd, so that for 0 ≤ i ≤ 12 mn, the i-th term and the (mn − 1 − i)-th
term cancel in the sum which is then 0. Note that

bi/mc + b(mn − 1 − i) /mc = bi/mc + bn − (1 + i) /mc


= n + bi/mc + b− (1 + i) /mc
= n + (bi/mc + b−i/m − 1/mc)
= n − 1, (2)

and similarly

bi/nc + b(mn − 1 − i) /nc = m − 1. (3)

Adding (2) and (3), we get (1). Suppose that m and n are both even, say
m = 2m0 and n = 2n0 . Then, for any positive integer i,
¹ º ¹ º ¹ º ¹ º ¹ º ¹ º
2i 2i 2i + 1 2i 2i 2i + 1
= = and = = .
m 2m0 2m0 n 2n0 2n0

5
Thus the sum, say S (m, n), is given by twice the sum over even indices:
1
mn−1
X 2 mn−1
X 0 0
S (m, n) = (−1)bi/mc+bi/nc = 2 (−1)b2i /mc+b2i /nc
i=0 i0 =0
1
2 mn−1
0 0
X 2mXn −1
bi0 /m0 c+bi0 /n0 c 0 0 0 0
= 2 (−1) =2 (−1)bi /m c+bi /n c
i0 =0 i0 =0
0 0 0 0
n −1
mX 2mXn −1
bi0 /m0 c+bi0 /n0 c 0 0 0 0
= 2 (−1) +2 (−1)bi /m c+bi /n c
i0 =0 i0 =m0 n0
0 0
n −1
mX
0 0 0 0 0 0 0 0
= 2S (m0 , n0 ) + 2 (−1)b(i +m n )/m c+b(i +m n )/n c
i0 =0
³ 0 0
´
0 0
= 2S (m , n ) 1 + (−1)n +m .
0 0
Now, if 1 + (−1)n +m = 0 ⇔ n0 + m0 is odd, in which case S (m0 , n0 ) = 0 and
S (m, n) = 0. Thus, S (m, n) = 0 ⇔ S (m0 , n0 ) = 0 ⇔ ... ⇔ the highest power
of 2 which divides m differs from the highest power of 2 which divides n.

B5. Let N be the positive integer with 1998 decimal digits, all of them 1; that
is,

N = 1111 · · · 11.

Find the thousandth digit after the decimal point of N.

Solution. Note that 9N = 1111 · · · 11 = 101998 − 1. Thus,


r
√ 101998 − 1 p p
N = = 13 101998 − 1 = 13 10999 1 − 10−1998
9
¡ ¢1
= 13 10999 1 − 10−1998 2

We have the binomial series


1
¡1 ¢ 1
¡1 ¢¡ ¢
1
1 2 2 −1 2 2 2 − 1 12 − 2 3
(1 − x) 2 =1− 2x + x − x +···
2! 3!
1
valid for |x| < 1. The remainder term in (1 − x) 2 = 1 − 12 x + R2 (x) satisfies
¯ µ ¶¯ ¯ ¯ 2
¯ d2 1 ¯ x2 ¯1 ¡1 ¢ − 2 ¯¯ x
3
|R2 (x)| ≤ max ¯¯ dt (1 − t) 2 ¯
¯ 2! = max ¯ − 1 (1 − t)
0≤t≤x ¯ 2 2 ¯ 2!
2
0≤t≤x
¯ ¯ 2
¯ − ¯ x
3
= ¯¯ 14 (1 − x) 2 ¯¯
2!

6
3

For x = 10−1998 , (1 − x) 2 ≤ 2 and so |R2 (x)| ≤ 14 x2 ≤ 14 10−2·1998 . Thus,
¯√ ¯ ¯
¯ ¡ ¢¯¯ 1 999 ¯ 1 ¡ ¢¯
1
¯ N − 3 10 999
1 − 2 x ¯ = 3 10 ¯(1 − x) 2 − 1 − 2 x ¯¯
1 ¯ 1

999 1 2 999−2·1998
≤ 1
3 10 4x ≤ 1
12 10 = 10−2998 .
Now
999
¡ ¢ 999
¡ ¢
1
3 10 1 − 12 x = 1
3 10 1 − 12 10−1998 = 13 10999 − 16 10−999
= .33 × 10999 − .166 × 10−999
999 999 ¡ ¢
= 3· · ·3.3· · ·3 + .333 − .166 × 10−999
999 999
= 3· · ·3.3· · ·3 + .166 × 10−999
999 999
= 3· · ·3.3· · ·3166.
Thus, the 1000-th digit to the right of the decimal is 1.
B6. √Prove that, for any integers a, b, c, there exists a positive integer n such
that n3 + an2 + bn + c is not an integer.
Solution. We try to write the assumed perfect square n3 + an2 + bn + c in the
¡ ¢2
form n3/2 + dn1/2 + f :
³ ´2
n3 + an2 + bn + c = n3/2 + d n1/2 + f
¡√ ¢3 √
= n3 + 2n2 d + 2 n f + nd2 + 2d nf + f 2 .
Choosing d = 12 a, and f = ±1, we then have, for n sufficiently large,
³ ´2 ³ ´2
n3/2 + 12 a n1/2 − 1 < n3 + an2 + bn + c < n3/2 + 12 a n1/2 + 1 .

If n is a perfect square, say n = m2 , then the extreme left and right are
perfect squares and there is only one perfect square between them, namely
¡ 3/2 1 ¢2
n + 2 a n1/2 . Hence, if n = m2 and n3 + an2 + bn + c is a perfect square,
then
³ ´2
n3 + an2 + bn + c = n3/2 + 12 a n1/2 = n3 + a n2 + 14 a2 n.
or
m6 + am4 + bm2 + c = m6 + a m4 + 14 a2 m2
or
bm2 + c = 14 a2 m2
For this to hold for all sufficiently large integers m, we must have c = 0 and
b = 14 a2 . Thus,
³ ´2 ³√ ³ a ´´2
n3 + an2 + bn + c = n3/2 + 12 a n1/2 = n n+ ,
2
which is not a perfect square, unless n is a perfect square.

You might also like